LSAT and Law School Admissions Forum

Get expert LSAT preparation and law school admissions advice from PowerScore Test Preparation.

 Administrator
PowerScore Staff
  • PowerScore Staff
  • Posts: 8917
  • Joined: Feb 02, 2011
|
#61035
Please post your questions below!
 litigationqueen
  • Posts: 12
  • Joined: Sep 23, 2020
|
#79207
Hi,

I chose A and the answer is C. I don't understand why my answer is wrong. This was my only incorrect answer from the Game so I believe my setup was good! Please advise, thanks.
 Rachael Wilkenfeld
PowerScore Staff
  • PowerScore Staff
  • Posts: 1358
  • Joined: Dec 15, 2011
|
#79304
Hi Litqueen,

Answer choice (A) is incorrect because we wouldn't be able to place enough Ks. Let's see why.

If we start with G in March, H must be in April because we can't have 3 Gs next to each other, and we can't have a K next to a G. So our first three would be GHG in March, April and May. That uses up all of our Gs, because we know that we have a G already in August, H has to be in Sept (as always in this game) and K is in October per the rules.

So now our set up looks like this GHG_ _ GHK _

This is where we can see the problem with answer choice (A). We have no where to put two more Ks! We can put one at the end of that chain, but where does the other one go? We can't put it in July or June without breaking the no GK/KG rule.

Hope that helps!
Rachael

Get the most out of your LSAT Prep Plus subscription.

Analyze and track your performance with our Testing and Analytics Package.